1. The medical assistant took the oral temperature of 42 patients on Monday, 37
patients on Tuesday, 65 patients on Wednesday, was off work on Thursday, and 56
patients on Friday. How many total thermometer probe covers were utilized by this
medical assistant this week at the clinic when taking patient temperatures?
2. The teenage patient grew % inch from January to March, ½ inch from March to May,
2 inches from May to September, and 1 % inches from September to December.
How many total inches did this patient grow this year?
3. An infant was born weighing 8 pounds and gained 2 pounds by his one-month
check-up, 2 pounds by his three-month check-up, and 4 more pounds by his
six-month check-up. How many pounds did this infant weigh by his 6-month
check-up appointment?
4. If 3 teaspoons = 0.5 fluid ounce and there are 4 ounces in a bottle of children's
Tylenol, how many 1-teaspoon doses are there in a bottle?

Answers

Answer 1

Answer:

1. The medical assistant utilized a total of 200 thermometer probe covers this week at the clinic when taking patient temperatures.

2. The teenage patient grew a total of 5 7/8 inches this year.

3. The infant weighed 14 pounds by his 6-month check-up appointment.

4. There are 24 1-teaspoon doses in a bottle of children's Tylenol.

Step-by-step explanation:

mark brainliest


Related Questions

a gardener uses a total of 61.5 gallons of gasoline in one month. of the total amount of gasoline, was used in his lawn mowers. how many gallons of gasoline did the gardener use in his lawn mowers in the one month? to get credit, you must show all of your work. answers only will be counted as incorrect (whether it is correct or not!)

Answers

The gardener used 61.5 gallons of gasoline in his lawn mowers in the one month.

Let's call the amount of gasoline used in the lawn mowers "x".

We know that the total amount of gasoline used is 61.5 gallons, so:

x + (the amount used for other things) = 61.5

We don't know how much was used for other things, but we do know that "of the total amount of gasoline" used, a certain percentage was used in the lawn mowers. Let's call that percentage "p".

"Of" means "times", so we can write:

p * 61.5 = x

Now we have two equations:

x + (the amount used for other things) = 61.5

p * 61.5 = x

We want to solve for x, so let's isolate it in the second equation:

p * 61.5 = x

x = p * 61.5

Now we can substitute that into the first equation:

p * 61.5 + (the amount used for other things) = 61.5

Simplifying:

p * 61.5 = 61.5 - (the amount used for other things)

p = (61.5 - the amount used for other things) / 61.5

We don't know the exact amount used for other things, but we do know that it's less than or equal to 61.5, so:

p = (61.5 - something) / 61.5

p = (61.5 - 0) / 61.5

p = 1

So all of the gasoline was used in the lawn mowers, and:

x = 1 * 61.5

x = 61.5

Know  more about percentage here:

https://brainly.com/question/29306119

#SPJ11

Drag each expression to its equivalent.
4y−3
9y
2+5y

Answers

Matching the algebraic expressions with their correct solutions gives:

8y - 6 - 4y + 3 → 4y - 3

y - 1 - 2 + 3y → 4y - 3

1 + y - 1 + 4y + 2 → 2 + 5y

4 + 5y - 3y - 4 + 3y + 2 → 2 + 5y

6 - 3y + 6y - 6 + 6y → 9y

How to solve Algebraic expressions?

Let us solve each of the algebraic expressions given:

1) 8y - 6 - 4y + 3

= 4y - 3

2) 6 - 3y + 6y - 6 + 6y

= 9y

3) y - 1 - 2 + 3y

= 4y - 3

4) 1 + 18y - 1 - 9y

= 9y

5) 1 + y - 1 + 4y + 2

= 5y + 2

6) 4 + 5y - 3y - 4 + 3y + 2

= 5y + 2

Read more about Algebraic Expressions at: https://brainly.com/question/4344214

#SPJ1

What are the domain and range of each relation? Drag the answer into the box to match each relation.

Answers

The domain and range for the relation are

for the graph: domain is [-3 3] and range is [-1 3]

domain is [-2 4] and range is [-3 0]

What is domain and range

The mathematics domain and range refer, respectively, to a function's input values as well as its output.

The set of possible input values that can be used for the function is called the domain or independant variable(s), while also comprising all necessary values for the calculation of appropriate results.

Conversely, the range or dependent variable(s) represents every conceivable result obtainable from specific sets of inputs within the domain. It essentially displays the function's abilities to produce an output value based on any given input it receives.

Learn more about domain and range at

https://brainly.com/question/2264373

#SPJ1

rewrite each proportion in fraction from. then find the value of each variable

×:8 = 9:24​

Answers

The value of the variable is 3

What is proportion?

Proportion can be defined as a method of comparing numbers in mathematics such that one is made equal to another.

Note that a fraction is described as the part of a whole

From the information given, we have that;

×:8 = 9:24​

To determine the fraction, we divide the numerator by the denominator, we have;

x/8= 9/24

Now, cross multiply the values

24(x) =9(8)

multiply the values, we have;

24x = 72

Now, make 'x' the subject

Divide both sides by the coefficient

x = 3

Learn more about proportion at: https://brainly.com/question/1781657

#SPJ1

a reproduction of a sculpture is made at a scale of 1:15 the reproduction is 13cm tall what is the height of the original sculpture in centimeters

Answers

The height of the original sculpture in centimeters is 195 cm

What is the height of the original sculpture in centimeters

From the question, we have the following parameters that can be used in our computation:

Scale = 1 : 15

Scale height = 13 cm

Using the above as a guide, we have the following:

13 cm : height = 1 : 15

Express the ratio as fraction

So, we have

height/13 cm = 15/1

Cross multiply

So, we have

height = 13 cm * 15/1

Evaluate

height = 195 cm/1

So, we have

height = 195 cm

Hence, the value of the actial height = 195 cm

Read more about scale at

https://brainly.com/question/29229124

#SPJ1

1. Please estimate a in a binomial distribution based on the number of events among n observations. n P(k events | a) = (%) *(1 – a)*-*, k = 0,1,2, ... , n

Answers

To estimate a in a binomial distribution, you can use the maximum likelihood estimation (MLE) method. Here are the steps:

1. Define the terms:
  - a: The probability of success in a single trial
  - n: The number of observations (trials)
  - k: The number of successful events among the n trials

2. Write the binomial probability function:
  P(k events | a) = (nCk) * (a^k) * (1 - a)^(n - k)

3. Calculate the likelihood function, which is the product of the binomial probability functions for all observed data points (for k = 0, 1, 2, ..., n).

4. Differentiate the logarithm of the likelihood function with respect to a (using logarithmic properties to simplify the expression) to obtain the first-order condition.

5. Set the first-order condition equal to zero and solve for a, which will give you the maximum likelihood estimate of a.

By following these steps, you can estimate a in a binomial distribution based on the number of events among n observations using the maximum likelihood estimation method.

Learn more about Binomial Distribution: https://brainly.com/question/31197941

#SPJ11

note: this is a multi-part question. once an answer is submitted, you will be unable to return to this part. what is the probability that bo, colleen, jeff, and rohini win the first, second, third, and fourth prizes, respectively, in a drawing if 50 people enter a contest and satisfying the following conditions? (enter the value of probability in decimals. round the answer to two decimal places.) winning more than one prize is allowed.

Answers

To find the probability that Bo, Colleen, Jeff, and Rohini win the first, second, third, and fourth prizes, respectively, in a drawing with 50 people, follow these steps:

1. Since winning more than one prize is allowed, the probability of Bo winning the first prize is 1/50.

2. Likewise, the probability of Colleen winning the second prize is also 1/50.

3. Similarly, the probability of Jeff winning the third prize is 1/50.

4. Finally, the probability of Rohini winning the fourth prize is 1/50.

5. Since these events are independent, we can multiply the probabilities together to find the overall probability of this specific  :

  Probability = (1/50) * (1/50) * (1/50) * (1/50)

6. Calculate the result:

  Probability ≈ 0.00000016

7. Round the answer to two decimal places:

  Probability ≈ 0.00

So, the probability that Bo, Colleen, Jeff, and Rohini win the first, second, third, and fourth prizes, respectively, in a drawing with 50 people is approximately 0.00.

To learn more about Decimal - brainly.com/question/30958821

#SPJ11

Susan wants to make aprons for cooking. She needs 1 1/2 yards of fabric for the front of the apron and 1/8 yards of fabric for the tie.
Part A: Calculate how much fabric is needed to make 3 aprons? Show every step of your work.
(5 points)
Part B: If Susan originally has 7 yards of fabric, how much is left over after making the aprons?
Show every step of your work. (5 points)
Part C: Does Susan have enough fabric left to make another apron? Explain why or why not. Please help me

Answers

Answer:

Sure, let's break down each part step by step.

Part A:

To calculate how much fabric is needed to make 3 aprons, we need to multiply the amount of fabric needed for one apron by 3.1 apron requires

1 1/2 yards of fabric for the front and 1/8 yards of fabric for the tie.1 1/2 yards + 1/8 yards = 15/8 yards (Adding fractions with a common denominator)

Now we can multiply the total fabric needed for one apron by 3 to get the fabric needed for 3 aprons:

3 * 15/8 yards = 45/8 yards (Multiplying by a whole number)

So, the total fabric needed to make 3 aprons is 45/8 yards.

Part B:

If Susan originally has 7 yards of fabric and she uses 45/8 yards to make 3 aprons, we can subtract the amount used from the original amount to find out how much fabric is left over.

7 yards - 45/8 yards = 56/8 yards - 45/8 yards (Subtracting fractions with a common denominator)

= 11/8 yards (Subtracting fractions)

So, after making the aprons, Susan will have 11/8 yards of fabric left over.

Part C:

To determine if Susan has enough fabric left to make another apron, we need to compare the amount of fabric left (11/8 yards) with the amount of fabric needed for one apron (1 1/2 yards + 1/8 yards = 15/8 yards).

Since 15/8 yards is greater than 11/8 yards, Susan does not have enough fabric left to make another apron. She is short by 4/8 yards (or 1/2 yard) of fabric.

Hope this helps! Let me know if you have any further questions.

Step-by-step explanation:

Coins are placed into a treasure chest, and each coin has a radius of 1.4 inches and a height of 0.0625 inches. If there are 230 coins inside the treasure chest, how many cubic inches of the treasure chest is taken up by the coins? Round to the nearest hundredth and approximate using π = 3.14.

0.38 in3
126.39 in3
353.88 in3
88.47 in3

Answers

Answer:

D

Step-by-step explanation:

The volume of a single coin is indeed:

Volume of a single coin = π × (radius)² × height

= 3.14 × (1.4 in)² × 0.0625 in

= 0.38465 in³ (rounded to the nearest hundredth)

Therefore, the total volume of 230 coins can be found by multiplying the volume of a single coin by the number of coins:

Total volume of 230 coins = 0.38465 in³/coin × 230 coins

= 88.47 in³ (rounded to the nearest hundredth, unrounded its 88.4695)

Hence, the answer is (D) 88.47 in³.

Square ABCD with vertices A(-7,5) B(-4,7) C(-2,4) and D(-5,2) 90 counterclockwise

Answers

The graph of a Square ABCD with vertices A(-7,5) B(-4,7) C(-2,4) and D(-5,2) is represent upper square and after 90° counterclockwise rotation the lower square represents ABCD in above figure.

A quadrilateral is a polygon that has number of four sides. This also implies that a quadrilateral has exactly four vertices, and exactly four angles. We have to graph a square with vertices A(-7,5), B(-4,7) ,C(-2,4) and D(-5,2) 90 counterclockwise. Now, steps to draw the square :

Each point having two coordinates, x-coordinate and y-coordinate. So, according to their values plot on graph. In last meet the all points to form a square. In above figure, upper square is normal square.

In case of rotating a figure of 90 degrees counterclockwise, each point of the figure has to be changed from (x, y) to (-y, x) and graph the rotated figure. So, now the vertices of square be A(-7,5), B(-4,7) ,C(-2,4) and D(-5,2). Now, draw the square for these point, lower square in above figure. Both graphs of square ABCD present in above figure.

For more information about square, visit :

https://brainly.com/question/25092270

#SPJ4

Complete question:

The above figure complete the question.

graph and label 9 and 10 and their given rotation about the origin. Give the coordinates of the images.

Square ABCD with vertices A(-7,5) B(-4,7) C(-2,4) and D(-5,2) 90 counterclockwise.

Find the sample size required to estimate a population mean with a given confidence level - Calculator Question The population standard deviation for the number of emails an individual gets each day is 94 emails. If we want to be 90% confident that the sample mean is within 17 emails of the true population mean, use a calculator to find the minimum sample size that should be taken

Answers

The minimum sample size that should be taken to be 90% confident that the sample mean is within 17 emails of the true population mean is 82.

To find the minimum sample size required to estimate a population mean with a given confidence level, we need to use the following formula:

n = (Z * σ / E)^2

where:
- n is the sample size
- Z is the Z-score corresponding to the desired confidence level (90% in this case)
- σ is the population standard deviation (94 emails)
- E is the margin of error (17 emails)

First, find the Z-score for a 90% confidence level. You can do this by checking a Z-table or using a calculator. For a 90% confidence level, the Z-score is approximately 1.645.

Now, plug the values into the formula:

n = (1.645 * 94 / 17)^2
n ≈ (153.63 / 17)^2
n ≈ 9.036^2
n ≈ 81.65

Since we cannot have a fraction of a person, round up to the nearest whole number. Therefore, the minimum sample size that should be taken to be 90% confident that the sample mean is within 17 emails of the true population mean is 82.

Know more about mean here:

https://brainly.com/question/1136789

#SPJ11

Which of the following could be trigonometric functions of the same angle?

Answers

The option that shows trigonometric functions of the same angle is:

Option C: cosY = 8 / 17, cotY = 8 / 15, secY = 17 / 8

How to Interpret trigonometric ratios?

The three primary trigonometric ratios are:

sin x = opposite/hypotenuse

cos x = adjacent/hypotenuse

tan x = opposite/adjacent

Here,

cosY = 8/17, cotY = 8/15, secY = 17 / 8

We know that in trigonometric ratios that:

cosY = 1 / SecY

Thus:

8 / 17 = 1 / secY

secY = 17 / 8

Now, using pythagoras theorem, we have:

P = √[17² - 8²]

P = 15

Thus:

Cot Y = 8 / 15

Read more about Trigonometric ratios at: https://brainly.com/question/13276558

#SPJ1

The heights of a sample of 15 students are recorded in the stemplot below.

A stemplot titled heights of students has values 59, 61, 62, 63, 63, 64, 65, 65, 66, 67, 67, 67, 67, 69, 73.

What is the mean height, in inches, of this sample?

65
65.2
66
67

Answers

Answer:

To find the mean height of the sample, we need to sum up all the values and divide them by the total number of values.

Sum of values = 59+61+62+63+63+64+65+65+66+67+67+67+67+69+73 = 964

Total number of values = 15

Mean height = sum of values / total number of values = 964/15 = 64.2666... ≈ 65.2

Therefore, the mean height, in inches, of this sample is approximately 65.2.

The answer is B.

expressing in standard /exact form, find all the complex numbers of z^3=sqrt3+isqrt5, using radians ,

Answers

The three complex cube roots of z^3 are:

z_1 = 2^(1/3) [cos(π/9) + i sin(π/9)]

z_2 = 2^(1/3) [cos(5π/9) + i sin(5π/9)]

z_3 = 2^(1/3) [cos(7π/9) + i sin(7π/9)]

First, we can find the modulus of the complex number as |z^3| = |√3+i√5| = √(3+5) = 2. We can also find the argument of the complex number as arg(z^3) = arctan(√5/√3) = π/3 - arctan(√3/√5).

Now, we can express the complex number in polar form as z^3 = 2(cosθ + i sinθ), where θ = π/3 - arctan(√3/√5).

Using De Moivre's theorem, we can find the cube roots of z as:

z_1 = 2^(1/3) [cos(θ/3) + i sin(θ/3)]

z_2 = 2^(1/3) [cos((θ+2π)/3) + i sin((θ+2π)/3)]

z_3 = 2^(1/3) [cos((θ+4π)/3) + i sin((θ+4π)/3)]

Simplifying further, we get:

z_1 = 2^(1/3) [cos(π/9) + i sin(π/9)]

z_2 = 2^(1/3) [cos(5π/9) + i sin(5π/9)]

z_3 = 2^(1/3) [cos(7π/9) + i sin(7π/9)]

Therefore, the three complex cube roots of z^3 are:

z_1 = 2^(1/3) [cos(π/9) + i sin(π/9)]

z_2 = 2^(1/3) [cos(5π/9) + i sin(5π/9)]

z_3 = 2^(1/3) [cos(7π/9) + i sin(7π/9)]

To learn more about complex visit:

https://brainly.com/question/21012874

#SPJ11

Divide the diffrence between 1200 and 700 by 5

Answers

Therefore, the quotient of the difference between 1200 and 700 divided by 5 is 100.

The slope of the secant line connecting two points on the graph of a function, f, is determined using the difference quotient. Just to refresh your memory, a function is a line or curve where there is just one y value and one x value.  The slope of secant lines may be calculated using the difference quotient.

Almost identical to a tangent line, a secant line traverses at least two points on a function. The slope of a secant line serves as the basis for the difference quotient formula. A function's difference quotient, y = f(x),

The difference between 1200 and 700 is: 1200 - 700

= 500

To divide this by 5, we simply divide 500 by 5:

500 ÷ 5 = 100

Learn more about difference visit: brainly.com/question/25433540

#SPJ4

The radius of cylinder A is 4 times the radius of cylinder B, and the height of cylinder A is 4 times the height of cylinder B. What is the ratio of the lateral surface area of A to the lateral surface area of B?​

Answers

Answer: The ratio of A's lateral surface area to B's lateral surface area is 16:1.

Step-by-step explanation: Let B's radius be x and the height be y. Then, the radius of A will be 4x and the height will be 4y.

As we know, the formula for the lateral surface area of a cylinder is

2[tex]\pi[/tex]rh.

So, the lateral surface area of A is 2[tex]\pi[/tex](4x)(4y)= 32[tex]\pi[/tex]xy

lateral surface area of B is 2[tex]\pi[/tex](x)(y)= 2[tex]\pi[/tex]xy

Ratio,

Lateral surface area of A/ Lateral surface area of B = [tex]\frac{32\pi xy}{2\pi xy}[/tex]

=[tex]\frac{16}{1}[/tex]

=16:1

What is length of side a given the following coordinates?
A (0,0), B(3,0), and C(2, 10).
A. 10.2
B. 79
C. 10.0
D. 3

Answers

Answer: A. 10.2

Step-by-step explanation: For this problem we have to create a second right triangle to find the length. You can apply the pythagorean theorem which continues to 10^2+2^2=c^2 which would get us 104. Then find the root of 104 which is equal to 10.2

An airline knows from experience that the distribution of the number of suitcases that get lost each week on a certain route is approximately normal with µ = 6.7 and σ = 3,5. What is the probability that the airline will lose at least 10 suitcases?

Answers

The probability that the airline will lose at least 10 suitcases in a week is 0.1723 or about 17.23%.

Given information:

µ = 6.7 (mean)

σ = 3.5 (standard deviation)

We need to find the probability of losing at least 10 suitcases in a week. We can use the normal distribution formula to solve this problem:

P(X ≥ 10) = 1 - P(X < 10)

To use this formula, we need to standardize the variable X to the standard normal distribution with mean 0 and standard deviation 1. We can do this using the following formula:

Z = (X - µ) / σ

Substituting the given values, we get:

Z = (10 - 6.7) / 3.5

Z = 0.943

Now, we can use a standard normal distribution table or calculator to find the probability of Z being greater than or equal to 0.943. The table or calculator will give us the probability of Z being less than 0.943, which we can then subtract from 1 to get the desired probability.

Using a standard normal distribution table, we find that P(Z < 0.943) = 0.8277.

Therefore, P(X ≥ 10) = 1 - P(X < 10) = 1 - P(Z < 0.943) = 1 - 0.8277 = 0.1723.

So, the probability that the airline will lose at least 10 suitcases in a week is 0.1723 or about 17.23%.

To learn more about probability visit:

https://brainly.com/question/15124899

#SPJ11

Is the sum of a rational and an irrational number, rational or irrational? For example, is 5 + pi rational or irrational? Explain why

Answers

This type of mathematics depends on the method used by the equation. For example, you stated 5+pi. It is REALLY simple. Just look at your answer from a calculator. If it says error, it’s probably irrational.

"in as much details as u can please thanxx,
9. (a) Study the variations of f(x) = r - In(1+x). (b) Study the variations of g(x) = (1 + x) In(1 + x) - 2. (c) Conclude that for all positive integer n, we have 1+1 x (1 + x)"

Answers

That kx^2 > 0 for x > 0, so we have 1+(k+1)x+kx^2 > 1+(k+1)x. Therefore, (1+x)^(k+1) > 1+(k+1)x, and the result follows by mathematical induction.

(a) To study the variations of f(x) = r - ln(1+x), we need to find the derivative of f(x) and analyze its sign.

The derivative is f'(x) = -1/(1+x), which is negative for all x > 0.

Therefore, f(x) is a decreasing function on (0, ∞).

Also, lim x→0 f(x) = r > -∞, and lim x→∞ f(x) = -∞.

Therefore, f(x) has a maximum at x = 0, which is r.

(b) To study the variations of g(x) = (1 + x) ln(1 + x) - 2, we need to find the derivative of g(x) and analyze its sign.

The derivative is g'(x) = ln(1 + x), which is positive for all x > -1.

Therefore, g(x) is an increasing function on (-1, ∞). Also, lim x→-1+ g(x) = -∞, and lim x→∞ g(x) = ∞.

Therefore, g(x) has a minimum at some point in (-1, ∞).

(c) To conclude that for all positive integer n, we have (1+x)^n > 1+nx, we can use mathematical induction.

For n = 1, we have (1+x)^1 = 1+x > 1+1x. Assume that (1+x)^k > 1+kx for some positive integer k. Then, for n = k+1, we have (1+x)^(k+1) = (1+x)^k * (1+x) > (1+kx) * (1+x) = 1+(k+1)x+kx^2.

Note that kx^2 > 0 for x > 0, so we have 1+(k+1)x+kx^2 > 1+(k+1)x. Therefore, (1+x)^(k+1) > 1+(k+1)x, and the result follows by mathematical induction.

To learn more about mathematical visit:

https://brainly.com/question/30569341

#SPJ11

9. Look at the graph below. If the object is rotated 180° about the z-axis, the coordinates for
Point A (-1, 2, 2) will be
1

Answers

The image of the point after it is rotated 180° about the z-axis is A' = (1, -2, -2)

Calculating the image of the point after the rotation

From the question, we have the following parameters that can be used in our computation:

Point A = (-1, 2, 2)

The rule of rotation is given as rotated 180° about the z-axis

Mathematically, this rule can be expressed as

(x, y, z) = (-x, -y, -z)

Substitute the known values in the above equation, so, we have the following representation

A' = (1, -2, -2)

Hence. the image of the point after the rotation is A' = (1, -2, -2)

Read more aboutt transformation at

https://brainly.com/question/4289712

#SPJ1

Helpppppppppppppppp?

Answers

start with 18 multiplied by 16 which is 288

then i believe other side length next to the 6 might be 2

so that would mean you do 6 multiplied by 12 and you subtract that fthe 288

so im pretty sure the answer is 276, but im not entirely sure

The given planes intersect in a line. Find parametric equations for the line of intersection. [Hint: The line of intersection consists of all points (x, y, z) that satisfy both equations. Solve the system and designate the unconstrained variable as t .]
x + 2y + z = 1, 2x+5y + 32 = 4

Answers

The parametric equations for the line of intersection are:
x = 61 - 5t
y = 2t - 30
z = t


To find the parametric equations for the line of intersection of the given planes, we first need to solve the system of equations:

1. x + 2y + z = 1
2. 2x + 5y + 32 = 4

Step 1: Solve for x from equation 1:
x = 1 - 2y - z

Step 2: Substitute x in equation 2 with the expression found in step 1:
2(1 - 2y - z) + 5y + 32 = 4

Now we can use elimination to solve for one variable. Let's eliminate y by multiplying the first equation by 5 and subtracting it from the second equation:

Step 3: Simplify and solve for y:
2 - 4y - 2z + 5y + 32 = 4
y - 2z = -30

Step 4: Designate z as the parameter t:
z = t

Step 5: Substitute z with t in the expression for y:
y = 2t - 30

Step 6: Substitute z with t in the expression for x:
x = 1 - 2(2t - 30) - t
x = 1 - 4t + 60 - t
x = 61 - 5t

Now we have the parametric equations for the line of intersection:
x = 61 - 5t
y = 2t - 30
z = t

Note that we can choose any value of z for the parameter t, since z is unconstrained.

Learn more about Parametric:

brainly.com/question/15585522

#SPJ11

You may need to use the appropriate appendix table or technology to answer this question.
A group conducted a survey of 13,000 brides and grooms married in the United States and found that the average cost of a wedding is $21,858. Assume that the cost of a wedding is normally distributed with a mean of $21,858 and a standard deviation of $5,800.
(a)
What is the probability that a wedding costs less than $20,000? (Round your answer to four decimal places.)
(b)
What is the probability that a wedding costs between $20,000 and $31,000? (Round your answer to four decimal places.)
(c)
What is the minimum cost (in dollars) for a wedding to be included among the most expensive 5% of weddings? (Round your answer to the nearest dollar.)
$

Answers

The probability that a wedding costs less than $20,000 is approximately 0.3745.

The probability that a wedding costs between $20,000 and $31,000 is approximately 0.6188.

The minimum cost for a wedding to be included among the most expensive 5% of weddings is approximately $31,229.

(a) To find the probability that a wedding costs less than $20,000, we need to standardize the value of $20,000 by subtracting the mean and dividing by the standard deviation:

z = (20000 - 21858) / 5800 = -0.32

We can then use a standard normal distribution table or technology to find the corresponding probability:

P(z < -0.32) ≈ 0.3745

Therefore, the probability that a wedding costs less than $20,000 is approximately 0.3745.

(b) To find the probability that a wedding costs between $20,000 and $31,000, we need to standardize both values and find the area between the corresponding z-scores:

z1 = (20000 - 21858) / 5800 = -0.32

z2 = (31000 - 21858) / 5800 = 1.58

Using a standard normal distribution table or technology, we can find the probabilities:

P(-0.32 < z < 1.58) ≈ 0.6188

Therefore, the probability that a wedding costs between $20,000 and $31,000 is approximately 0.6188.

(c) To find the minimum cost for a wedding to be included among the most expensive 5% of weddings, we need to find the z-score that corresponds to the 95th percentile of the standard normal distribution. We can use a standard normal distribution table or technology to find this value:

z = invNorm(0.95) ≈ 1.645

We can then use the formula for standardizing a value to find the minimum cost:

z = (x - 21858) / 5800

Solving for x, we get:

x = z(5800) + 21858

x = 1.645(5800) + 21858

x ≈ 31229

Therefore, the minimum cost for a wedding to be included among the most expensive 5% of weddings is approximately $31,229.

To learn more about technology visit:

https://brainly.com/question/9171028

#SPJ11

How many tons are equal to 36,000 pounds?
O 1,800 tons
O 180 tons
O 18 tons
08 tons

Answers

The answer is 18 tons.

Diane also has a number of nonfiction books. Of those books, 28% are hardcover, 22% are reference books, and 13% are hardcover reference books. Diane will select a nonfiction book at random. Let the event that the selected book is a hardcover be H and the event that it is a reference book be R. What is the probability that it is neither a hardcover nor a reference book.

Answers

The probability that it is a hardcover or a reference book  will be 0.37. in other words, the probability is the number that shows the happening of the event.

What is probability?

It is defined as the ratio of the number of favorable outcomes to the total number of outcomes,

Event H; Selected base is hardcover

Reselected book is a reference book

P(H) = 0.28

P(R) = 0.22

P(H∩R)=0.13

The probability that it is a hardcover or a reference book;

P(H∪R)=P(H)+P(R)-P(H∩R)

P(H∪R)=0.28+0.22-0.13

P(H∪R)=0.37

Hence, the probability that it is a hardcover or a reference book will be 0.37.

Learn more about probability on https://brainly.com/question/31171712

#SPJ1

Directions: There are 11 questions in 5 pages. No credit will be given without sufficient work 1. Let Z be a standard normally distributed random variable. Find: a. P(Z S 2.32) b. P(Z 2-1.56) c. P(-1.43 SZ 52.47) d. Find : so that P(-:* SZS :) 0.99

Answers

As given below find the suitable option which gives you the answer for the question. "There are 11 questions in 5 pages. No credit will be given without sufficient work 1. Let Z be a standard normally distributed random variable."

1. Let Z be a standard, normally distributed random variable.
a. P(Z ≤ 2.32)
To find this probability, you need to use the standard normal distribution table (also known as the Z-table) to look up the value corresponding to Z = 2.32. The value you find in the table is the probability P(Z ≤ 2.32).
b. P(Z ≥ -1.56)
To find this probability, first look up the value corresponding to Z = -1.56 in the standard normal distribution table. This value represents P(Z ≤ -1.56). Since we want P(Z ≥ -1.56), we need to find the complement, which is 1 - P(Z ≤ -1.56).
c. P(-1.43 ≤ Z ≤ 2.47)
To find this probability, look up the values corresponding to Z = -1.43 and Z = 2.47 in the standard normal distribution table. The difference between these two values will give you the probability P(-1.43 ≤ Z ≤ 2.47).
d. Find z* so that P(-z* ≤ Z ≤ z*) = 0.99
To find the z* value, you need to look up the value in the standard normal distribution table that corresponds to the area of 0.995 (since 0.99 is the area between -z* and z*, and each tail contains 0.005). Once you find the value in the table, look at the corresponding Z value. This value will be your z*.

Learn more about the standard: https://brainly.com/question/24298037

#SPJ11

Tickets to a play cost $6.50 each. Write an equation
for the total cost of 12 tickets plus a $7.50 fee for
large groups.

Answers

The equation for the total cost of 12 tickets plus a $7.50 fee for large groups is:

12($6.50) + $7.50 = $90

PLEASE HELP ITS URGENT I INCLUDED THE GRAPH AND WROTE THE PROBLEM DOWN ITS THE IMAGE I HAVE ATTACHED!!!

Answers

Answer:

Ive attached a picture

Step-by-step explanation:

Determine the product of 15/6 and 1.2

Answers

Answer:

3

Step-by-step explanation:

Other Questions
find the number of lamps should be manufactured annually write a program that stores the maximum of three values. the values are stored in $s0, $s1, and $s2. store the result in $s3. ex: if the values of $s0, $s1, and $s2 are initialized in the simulator as: registers data $s0 5 $s1 9 $s2 8 the result is stored in $s3: registers data $s0 5 $s1 9 $s2 8 $s3 9 note: use the ' ' button under the registers display to initialize register values for $s0, $s1, and $s2. 7. the rock and his wife are warned that the road is out ahead. they come to a place where the earth has been displaced. there are two types of faults here. name one of them and explain your reasoning. (if you can name them both, that would be awesome!) what are 6 symptoms of sedative, hypnotic, and/or antianxiety medication intoxication? (SIUNIC) g what is the ratio of nondedicated collections funds to total revenue? (2) what is the quick ratio (use known current liabilities in the calculation)? (3) what is the capital asset condition as calculated by the ratio of accumulated depreciation to the cost of depreciable capital assets? Which sentence has a compound word? (1 point)O The kids all ran to the playground during recess.O The science test was challenging.O Chocolate cake is the most popular dessert.O We learned about geography today. Language art. Michael solved the given problem. Determine whether Michael's work is correct. If not, which step is incorrect?ResponsesA Step 1B Step 4C Step 3D Step 2 The equilibrium constant, K, for the synthesis of ammoniaN(g) + 3H(g) 2NH3(g)is approximately 44005 at 127 C.What is the value of K, for the decomposition of ammonia at 127 C, as described by the following equation?NH3 (g) 1/2N(g) + 3/2 H(g) Which device is used to transfer the contents of one syringe to another?Select one:Hub-to-hub capLuer-to-luer connectorRubber syringe capsVented needle A client returns to the clinic for follow-up treatment following a skin biopsy of a suspicious lesion performed 1 week ago. The biopsy report indicates that the lesion is a melanoma. The nurse understands that melanoma has which characteristics? Select all that apply. Individuals who have mastered the skills specific to human resource management have the competencies they need for success in this field. O True O False a plaintiff has sued a defendant, alleging that she was run over by a speeding car driven by the defendant/ Item 13A student is applying to two different agencies for scholarships. Based on the students academic record, the probability that the student will be awarded a scholarship from Agency A is 0. 55 and the probability that the student will be awarded a scholarship from Agency B is 0. 40. Furthermore, if the student is awarded a scholarship from Agency A, the probability that the student will be awarded a scholarship from Agency B is 0. 60. What is the probability that the student will be awarded at least one of the two scholarships? true or false?if a woman is diagnosed with bacterial vaginosis, the partner should be notified and treated simultaneously Now, using the information you have learned, write your letter to the president. Start your letter by typing dear President Truman in the space to the right. Then, in your first paragraph, state who you are and why you were writing. Next, state what your position is on, dropping the bomb, and provide one reason that supports it. Then provide a second reason. Daniel, who is three years old, has been found to be highly allergic to peanuts, milk, and eggs. What measures and/or policies will you put in place in your early childhood education environment to see that he is safe and never exposed to any of these products? What foods would you serve to ensure that all children in your care are meeting their nutritional needs while still keeping Daniel safe? Let tans = -5 and 3x < < 5x/2. Find the exact value of the following. A) tan(2)b) cos(2)c) tan(/2) Osteoporosis is a disease that weaken bones, and it runs in Angelicas family. What would be the BEST form of exercise for Angelica to start doing to address this potential problem? overeating, drinking, smoking, other substances, withdrawing, yelling, and fighting are examples of ________________ coping mechanisms When considering a change in momentum what two variables must you consider?